The sum of 4 44 consecutive integers is 198 198198. What is the fourth number in this sequence?.

Answers

Answer 1

With the sequence having sum of four consecutive integers as 198 , the fourth integer in the sequence is calculated to be 51.

Let's call the first integer in the sequence "x". Since the integers are consecutive, the next three integers in the sequence would be "x + 1", "x + 2", and "x + 3". The sum of these four integers can be represented as:

x + (x + 1) + (x + 2) + (x + 3) = 198

Expanding and simplifying the left side of the equation, we get:

4x + 6 = 198

Subtracting 6 from both sides of the equation, we get:

4x = 192

Dividing both sides of the equation by 4, we get:

x = 48

So the first integer in the sequence is 48. To find the fourth integer in the sequence, we add 3 to the first integer:

x + 3 = 48 + 3 = 51

So the fourth integer in the sequence is 51.

To learn more about sequence click on,

https://brainly.com/question/2073686

#SPJ4


Related Questions

Shawn's mortgage balance of $263,000 is
compounded daily at a rate of 4.075%
annualized interest. If no payments are made
for 6 months, what will the balance be after
this time has passed?

Answers

The balance amount when compounded daily at a rate of 4.075% after 6 months is  $269,320.60.

What is meant by compound interest?

The interest you earn on interest is known as compound interest. Compound interest, also known as interest on principal & interest, is the practice of adding interest to the principal amount of a loan or deposit. Compound interest is calculated for an amount based on both the principal and cumulative interest. The major distinction between compound and simple interest is this. If we examine our bank statements, we will typically see that our account is credited with interest on a yearly basis. Even while the principal remains the same, the interest changes annually. We can observe that interest rises over time. Thus, we can infer that the interest the bank charges is compound interest rather than simple interest.

Given,

Principal amount P = $263,000

Rate of interest r = 4.075%

Time period t = 6 months = 0.5 years

Final amount =  [tex]A = P(1 + \frac{r}{n})^{nt}[/tex]

where n = 365

A = [tex]P(1 + \frac{r}{n})^{nt} =263000(1 + \frac{0.04075}{365})^{365*0.5}[/tex]  = $269,320.60

Therefore the balance amount when compounded daily at a rate of 4.075% is  $269,320.60.

To learn more about compound interest, follow the link.

https://brainly.com/question/28020457

#SPJ1

Solve for X (pls give explanation as well)
2x + 6
70°

Answers

The sum of all the angles of triangle is 180°

We have the following angles:

70°90°2x+6

The sum of all these angles will be equal to 180°

[tex] \sf \: 70 + 90 + 2x + 6 = 180[/tex]

Add all the constants

[tex] \sf166 + 2x = 180[/tex]

Move the constant to the right hand side and change its sign

[tex] \sf2x = 180 - 166[/tex]

Subtract the numbers

[tex] \sf2x = 14[/tex]

Divide both sides by 2

[tex] \sf2x \div 2 = 14 \div 2[/tex]

[tex] \boxed{ \tt \: x = 7}[/tex]

Now,

The given angle is 2x+6, So put the value of x in this and find the value of angle

2x+62×7+614+620°

1. Interpret the domain restrictions on the demand and the supply functions. What do they represent in context of the scenario?

2. Suppose that in her first month Yaseen is able to create 15 kits. How much should she charge for each of these kits based on her supply function? How much should she charge for each of these kits based on her demand function? Show your work.

3. In her first month of business, Yaseen sells out of her kits in 3 days. In her second month of business, she decides to supply 60 kits. How much should she charge for each of these kits based on her supply function? How much should she charge for each of these kits based on her demand function? Show your work.

4. If Yaseen creates 60 kits in her second month of business and offers them for sale at $66 each, do you think she will sell all of them? Why or why not?

5. What family of functions do f(x) and g(x) belong to? How do you know?

6. If you were to graph two quadratic functions on the same xy-plane, how many intersection points could there be? Use examples to explain your thinking.

7. Use algebraic methods to find where Yaseen’s demand and supply functions intersect. What does this point(s) represent in context of this problem? Show all your work.

8. Confirm your answer from question 7 by using Desmos to create a single graph that shows both () and () with their domain restrictions. Provide a screenshot of your graph with the intersection point(s) marked.

Answers

To create 15 kits, Yaseen charge for each of these kits based on her supply function is $17.50.

What is the function?

Functions are the fundamental part of the calculus in mathematics. The functions are the special types of relations. A function in math is visualized as a rule, which gives a unique output for every input x.

Yaseen can produce 15 kits, the cost per kit she should charge is

p = 10 + 0.5q

p = 10 + 0.5(15) (15)

p = $17.50

She can produce 15 kits, the cost per kit she should charge is:

p = 30 - 0.5(15) (15)

p = $22.50

She can provide 60 kits, the cost per kit should be as follows:

p = 10 + 0.5q

Here, q=60

p = 10 + 0.5(60)

p = $40

She can produce 60 kits, the cost per kit should be set at:

p = 30 - 0.5(60) (60)

p = $15

Therefore, to create 15 kits, Yaseen charge for each of these kits based on her supply function is $17.50.

To learn more about the function visit:

https://brainly.com/question/28303908.

#SPJ1

Franco has a hat with 9 blue marbles, 4 yellow marbles, and 12 green marbles. He designs a binomial experiment by drawing a marble from the hat, recording whether the marble is green, and then laying the marble aside. He then repeats the process seven times. Which statement must be true?
The experiment is not a binomial experiment because there is not a fixed number of drawings.
The experiment is not a binomial experiment because the probability of choosing a green marble is the same for each drawing.
The experiment is not a binomial experiment because the probability of choosing a green marble is not the same for each drawing.
The experiment is not a binomial experiment because there are only two possible outcomes for each drawing.

Answers

The experiment is not a binomial experiment because the probability of choosing a green marble is not the same for each drawing.

Answer: c

Step-by-step explanation:

In the synthetic-division problem shown below, what number belongs in the
place of the question mark?
2 1 -2 -8 5
2
?
10☐☐.

Answers

The solution is, the remainder is 9.

What is division?

Division is the process of splitting a number or an amount into equal parts.

Division is one of the four basic operations of arithmetic, the ways that numbers are combined to make new numbers. The other operations are addition, subtraction, and multiplication.

here, we have,

find the remainder in the synthetic division

1         4       6        -1

Take down the first number 4 as it is. then multiply it with divisor 1 and put it below 6. then add it and do the same till we get remainder

1         4       6        -1

        0        4        10

       --------------------------------

        4        10       9 -> Remainder

So the remainder is 9.

To learn more on division click:

brainly.com/question/21416852

#SPJ9

HELPPPP ASAP 25 POINTS AN DID WILL MARK YOU AS BRAINLIEST!!!!Jack, Art, Fran, and Megan work as volunteers at the local kennel, Jack gives the dogs baths eve 4 days; Art cleans out cages every 6 days; Fran feeds the animals in Section Bevery 2 days, and Megan helps the receptionist every 3 days. How many times in 12 weeks will all 4 helpers be at the clinic on the same day? Show all work and explain how you got your answer.​

Answers

Answer:

Below

Step-by-step explanation:

I will assume they are all there on day 1

 then the LCM of  4  6  2  and 3   is 12 days

    every 12 days they are there together

       12 weeks = 12 x 7 = 84 days

                 84 / 12 = 7 times

day   1    13     25    37    49    62    74

SWIX (avio scab In circle O above, the length of arc BC is 2 cm, mZBOC = 32°, and m¿AOB = 96°. What is the length of arc AB?​

Answers

The length of the arc AB is 6 cm when BC is 2 cm, ∠BOC = 32°, and ∠AOB = 96°.

What is the arc length of a circle?

The arc length is the portion of the circumference of a circle. It is directly proportional to the central angle formed by an arc.

The center of the circle is O, where BC = 2 cm, ∠BOC = 32°, and ∠AOB = 96°. Draw the diagram as per the given data.

As 96° = 3×32°, it follows that ∠AOB = 3×∠BOC.

As the central angle formed by an arc is proportional to its length. It follows that AB = 3×BC.

Since, BC = 2 cm, it can be got that AB = 3×2 cm = 6 cm.

Therefore, the required answer is 6 cm.

To learn more about  arc length visit this link

https://brainly.com/question/29886215

#SPJ9

How many packets,each of dimensions 16 cm by 9 cm by 4 cm can be placed in a cubical box of side 1.2m long ?

Answers

The number of packets of dimensions 16 cm by 9 cm by 4 cm in a cubical box will be 3,000.

What is the volume?

Volume is a measurement of three-dimensional space that is utilized. It is frequently mathematically quantified using SI-derived units or different imperial or US traditional units. The concept of length is linked to the notion of capacity.

The volume of each packet is given as,

v = 16 x 9 x 4

v = 576 cubic cm

The volume of the cubical box is given as,

V = (1.2 x 100) x (1.2 x 100) x (1.2 x 100)

V = 120 x 120 x 120

V = 1,728,000 cubic cm

Then the number of packets is given as,

n = V / v

n = 1,728,000 / 576

n = 3,000

The number of packets of dimensions 16 cm by 9 cm by 4 cm in a cubical box will be 3,000.

More about the volume link is given below.

https://brainly.com/question/1578538

#SPJ9

If 0° ≤ Θ≤ 360°, then find angle(s) Θ.
tanΘ=1

Answers

Answer: Θ = [tex]45[/tex]

Step-by-step explanation:

tan x=sinx/cosx

and tan x is negative in the first and third quadrants.

tan x=1 for 45 degrees so tan x=-1 in the first quadrant 135 degrees and the third quadrant= 315 degrees

ILL GIVE BRAINLEST FOR SHOWING WORK, rail train scale of 1:87 model:11 in actual:?

Answers

The actual train is 957 inches long.

What is a Ratio?

A ratio indicates the number of times one number contains another. The ratio is defined as the comparison of two quantities of the same units that indicates how much of one quantity is present in the other quantity.

Given that, the ratio of the model train to the actual train is 1 : 87.

The model train is 11 inches long, therefore,

1=11

Then, the value of 87 is:

87 = 87×11

   = 957

Hence, the actual train is 957 inches long.

To learn more about ratios, click here:

brainly.com/question/13419413

#SPJ9

A hotel had 45 guests call for room service, and 200 who did not. What percentage of the guests called for room service?

Answers

Answer:

Step-by-step explanation:

Number of guests who called for room service = 45

Total number of guests = 45 + 200 = 245

Percentage of guests who called for room service = (45/245) x 100% ≈ 18.4%

Therefore, approximately 18.4% of the hotel's guests called for room service.

About 18.4%

Explanation:

Add the number of people who called for room service which is 45 to the number of people who didn’t which is 200

200+45 = 245

(45/245) x 100% = 18.36%

Which rounds up to 18.4%

Compare / Contrast Exponential Equations

1.) 2 (1/5)x-5

2.) 3 (1/2)x-3

Answers

The main difference between the two equations is the rate at which the graphs increase (determined by the horizontal scaling factor) and the vertical position of the graph (determined by the vertical scaling factor and translation).

How did we get this assertion?

Both exponential equations are in the form y = ab^x - c, where a is the vertical scaling factor, b is the horizontal scaling factor (also known as the growth factor), and c is the vertical translation. The difference between the two equations lies in the values of the parameters a, b, and c.

The first equation, 2 (1/5)x-5, has a vertical scaling factor of 2, a horizontal scaling factor of 1/5, and a vertical translation of -5. The graph of this equation will increase at a slower rate than the graph of the second equation.

The second equation, 3 (1/2)x-3, has a vertical scaling factor of 3, a horizontal scaling factor of 1/2, and a vertical translation of -3. The graph of this equation will increase at a faster rate than the graph of the first equation.

In summary, the main difference between the two equations is the rate at which the graphs increase (determined by the horizontal scaling factor) and the vertical position of the graph (determined by the vertical scaling factor and translation).

learn more about exponential equation: https://brainly.com/question/2456547

#SPJ1

Bailey is asked to draw a circle. She is given the center and two points on the circle, but she isn't told which point is which. The points are (-6, 3), (-15, 3), and (3,3). Bailey draws an accurate circle from this information. What is the equation of the circle she drew?
O (= - 6)3 + (3 - 3)? = 3
O (= - 3)? + (y - 3)? = 3
O (* + 6)3 + (y - 3)? = 81
О (= - 3)3 + (y - 3)? = 81
〇(c+15)+(- 3) = 81
O (= - 6)? + (3 - 3)? = 81p

Answers

The Equation of Circle is (x+ 6)² + (y - 3)² = 9².

What is Equation of Circle?

The equation for a circle is given by:

(x-h)² +(y-k)² = a²

Where (h,k) is the center and a is the radius of the circle.

Given:

We have the points A(-6, 3), B(-15, 3), and C(3,3).

Using Distance Formula

AB = √(-15 + 6)² + (3-3)²

AB = √9²

AB= 9 unit

BC= √(3 + 15)² + (3-3)²

BC = 18 unit

and, AC = √(3+6)² + (3-3)²

AC = 9 unit

As, AB = AC then A has to be the center (-6, 3) with radius 9 unit.

Thus, the Equation of Circle is

(x+ 6)² + (y - 3)² = 9²

Learn more about Equation of Circle here:

https://brainly.com/question/29288238

#SPJ1

4, 8, 16, 32, 64

Write the explicit form of the sequence above.

Answers

Answer:

I think its

Step-by-step explanation:

2^(n+1) is the explicit form

4v + 9 x 7? step by step

Answers

4v + (9 x 7)= 4v +56

I believe is like

First multiply 9x7 =63
So is going to be 4v + 63
Cz you cant add 4v to 63 bc 63 doesn’t have a v


Maya made fruit punch for a party. She mixed 2 gallons of orange juice, 5 quarts of pineapple juice, 4 pints of cranberry juice, and 6 cups of apple juice. How many quarts did she make in all?


Answers

Answer:

To add different quantities of liquids with different units of measurements, we need to convert them all to the same unit of measurement. In this case, let's convert everything to quarts since that is what we want the final answer to be in.

2 gallons = 8 quarts (since 1 gallon = 4 quarts)

5 quarts of pineapple juice = 5 quarts (no conversion needed)

4 pints of cranberry juice = 8 cups = 2 quarts (since 1 pint = 2 cups and 1 quart = 4 cups)

6 cups of apple juice = 1.5 quarts (since 1 cup = 0.25 quarts)

Now we can add up all the quantities:

2 + 5 + 2 + 1.5 = 10.5

Maya made 10.5 quarts of fruit punch in all

convert 1,235 to three decimal places

Answers

Answer: la response est 1,235 je pense

Step-by-step explanation: trois decimal veux dire trois chiffre après la virgule donc on decale la virgule de 3 vers la gauche

image is the question

Answers

Answer:

350x + 200y - 4200 = 0

x intercept is (12,0)

y intercept (0,21)

steps:

ii

350x + 200y - 4200 = 0

Therefore, the equation in standard form that models the number of funnel cakes, x, and the number of deep-fried Oreos, y, Francis can buy if he spends $42.00 is:

350x + 200y - 4200 = 0

To find the x-intercept, we set y to zero and solve for x:

350x + 200(0) - 4200 = 0

350x = 4200

x = 12

Therefore, the x-intercept is 12. This means that if Francis only buys deep-fried Oreos (y = 0), he can buy 12 funnel cakes with $42.00.

Answer:

350x + 200y - 4200 = 0

x intercept is (12,0)

y intercept (0,21)

steps:

3.50x + 2.00y = 42.00

3.50x + 2.00y - 42.00 = 0

Multiplying both sides by 100 to eliminate the decimals, we get:

350x + 200y - 4200 = 0

x-intercept means what is x when y is 0

350x + 200(0) - 4200 = 0

350x = 4200

x = 12

y-intercept means what is y when x is 0

350(0) + 200y - 4200 = 0

200y = 4200

y = 21

So the x-intercept is (12, 0) and the y-intercept is (0, 21).

ChatGPT

To find the y-intercept, we set x to zero and solve for y:

350(0) + 200y - 4200 = 0

200y = 4200

y = 21

Therefore, the y-intercept is 21. This means that if Francis only buys funnel cakes (x = 0), he can buy 21 deep-fried Oreos with $42.00.

So the x-intercept is (12, 0) and the y-intercept is (0, 21).

ChatGPT

What are the coordinates of the point on the directed line segment from (-1,-9) to (8,9) that partitions the segment into a ratio of 4 to 5

Answers

Answer: To find the point on the directed line segment that partitions it into a ratio of 4 to 5, we can use the formula for a point on a line given two endpoints and a ratio. The formula is:

(x, y) = (1 - t) * (x1, y1) + t * (x2, y2)

where (x1, y1) and (x2, y2) are the coordinates of the endpoints of the line segment, and t is a value between 0 and 1 that determines the location of the point along the line.

In this case, (x1, y1) = (-1, -9) and (x2, y2) = (8, 9), and we want the point that partitions the line segment into a 4 to 5 ratio, so:

t = 4 / (4 + 5) = 4 / 9

Plugging in the values into the formula:

x = (1 - t) * x1 + t * x2 = (1 - 4/9) * (-1) + (4/9) * 8 = (5/9) * 8 - (4/9) * 1 = 44/9 - 4/9 = 40/9

y = (1 - t) * y1 + t * y2 = (1 - 4/9) * (-9) + (4/9) * 9 = (5/9) * 9 - (4/9) * (-9) = 45/9 + 36/9 = 81/9

So the point that partitions the line segment into a 4 to 5 ratio is (40/9, 81/9).

Step-by-step explanation:

PLEASE HELP ASAP!!

4. Given f(x) = x^2 -3x -4 and g(x) = 2x^2 -5x – 12.
5. Describe how to find:
a. (g-f)(x)
b. (f/g)(x)
6. Would both of these functions have the same
domain and range? Explain why or why not.

Answers

Value of composite functions (g-f)(x)  and (f/g)(x) are x² - 2x - 8 and

(x+1)/(2x - 3) respectively.both of these functions don't have the same

domain and range.

What is function?

Function, in mathematics, an expression, rule, or law that defines a relationship between one variable (the independent variable) and another variable (the dependent variable).

Given functions,

f(x) = x² - 3x - 4

g(x) = 2x² - 5x - 12

1. (g - f)(x)

= g(x) - f(x)

= 2x² - 5x - 12 - (x² - 3x - 4)

= 2x² - 5x - 12 - x² + 3x + 4)

= x² - 2x - 8

x is defined for all real numbers

Domain = (-∞, ∞)

let y = x² - 2x - 8

x = √y + 2(x) - 8

Function is defined for y ≥ 9

Range is {y∈R , y ≥ 9}

2. (f/g)(x)

= f(x)/g(x)

= (x² - 3x - 4)/(2x² - 5x - 12)

= (x² - 4x + x - 4)/(2x² - 8x + 3x - 12)

= (x(x - 4) + 1(x - 4))/(2x(x - 4) + 3 (x - 4))

= (x+1)(x-4)/(2x+3)(x-4)

= (x+1)/(2x - 3)

function is defined for x ≠ 3/2, Domain is {x ∈ R : x ≠ 3/2}

Let y = (x+1)/(2x - 3)

y.2x - 3y = x + 1

-3y - 1 = x - 2xy

x = (-1 - 3y)/(1 - 2y)

function is defined for y ≠ 1/2, Range is {y ∈ R : y ≠ 1/2}

Domain and range are not same for both functions.

Hence,  Value of composite functions (g-f)(x) = x² - 2x - 8 and

(f/g)(x) = (x+1)/(2x - 3), Domain and range is not same for both functions.

Learn more about function here:

https://brainly.com/question/12431044

#SPJ1

Two parallel lines are cut by a transversal as shown below.
Suppose m1=38°. Find m6 and m7.

Please help I don’t know how to do this
(See attached image)

Answers

Answer:

m∠6 = 142°

m∠7 = 38°

Step-by-step explanation:

You will be using postulates and theorems to solve this question.

Two parallel lines are cut by a transversal          (Given)

- m∠1 = 38°                         (Given)

- m∠1 ≅ m∠3                      (Vertical Angles Theorem)

  ∴ m∠3 = 38° (Definition of Vertical Angles Theorem)

- m∠3 ≅ m∠5                     (Alternate Angles Theorem)

 m∠3 = 38° ∴ m∠5 = 38° (Definition of Alternate Angles Theorem)

- m∠5 ≅ m∠7                     (Vertical Angles Theorem)

 m∠5 = 38° ∴ m∠7 = 38° (Definition of Alternate Angles Theorem)

m∠7 + m∠6 = 180°            (Corresponding Angles Postulate)

m∠7 = 38°                          (Given above)

Plug in 38 for m∠7 and solve for m∠6:

(38) + m∠6 = 180

Isolate m∠6. Note the equal sign, what you do to one side, you do to the other. Subtract 38 from both sides of the equation:

m∠6 + 38 = 180

m∠6 + 38 (-38) = 180 (-38)

m∠6 = 180 - 38

m∠6 = 142°

-

Answers:

m∠6 = 142°

m∠7 = 38°

~

Learn more about doing proofs, here:

https://brainly.com/question/28613847

2(x+4)=-6 what is x?

Answers

Answer: x = -7


explanation:


2(x+4)= -6

2x+8= -6

-8 -8

2x -14

Divide both sides by 2

X = -7

Answer:x=-7 I hope this helps

Step-by-step explanation:

2(x+4)=-6

Divide both sides of the equation by 2

X+4=-3

Move the constant to the right-hand side and change its sign

X=-3-4

Calculate the difference which means subtract-3 minus 4 which give you -7

X=-7

And that how you solve for X

Factor out the gcf

10x^3+10x^2

Answers

The factorization of the given polynomial is 10x²(x+1).

What are factors of polynomial?

The factors are the polynomials which are multiplied to produce the original polynomial.

The given polynomial is 10x³+10x².

Here, factors of the terms 10x³ and 10x² are

10x³= 2×5×x×x×x

10x²= 2×5×x×x

GCF of the terms is 2×5×x×x=10x²

10x²(x+1)

Therefore, the factorization is 10x²(x+1).

To learn more about the factorization of polynomial visit:

https://brainly.com/question/16789195.

#SPJ9

How to find the variance of uniform distribution?

Answers

We can find the variance of a uniform distribution, using the following formula: Variance = (b - a)² / 12. Here: a is the lower limit of the uniform distribution, b is the upper limit of the uniform distribution.

The formula for the variance of a uniform distribution is based on the fact that the variance of a continuous uniform distribution is equal to the square of the range (i.e., the difference between the upper and lower limits) divided by 12.

For example, let's say we have a uniform distribution between 0 and 10. To find the variance, we would use the formula:

Variance = (10 - 0)² / 12

Variance = 100 / 12

Variance = 8.33

Therefore, the variance of a uniform distribution between 0 and 10 is 8.33.

Learn more about uniform distribution here: brainly.com/question/22209943

#SPJ4

If (a,-8) is a solution to the equation-a=4b-7, what is a

Answers

thus, a = -39 is the solution to the equation -a = 4b-7 when (a, -8) is a solution

What's equation?

An equation is a fine statement that asserts the equivalency of two expressions. It generally consists of two sides, with an equal sign between them. Equations are used to describe connections between variables, and to break problems in colorful fields, including mathematics, wisdom, engineering, and economics.

Given by the question.

the equation given is -a = 4b - 7.

We are also told that (a, -8) is a solution to this equation. This means that if we substitute a for x and -8 for y, the equation will be true.

Substituting the given values, we get:

a = 4b - 7 (Given equation)

a = 4(-8) - 7 (Substituting b = -8, since (a, -8) is a solution)

a = -32 - 7 (Simplifying)

a = -39 (Final answer)

To learn more about equation:

https://brainly.com/question/24283639

#SPJ1

HELPPPP ASAP 25 POINTSSS AND I WILL MARK YOU BRAINLIEST2. For a school Project, Ron recorded how much TV he watched in one weekend and what kinds of programs he watched. This chart shows the time he watched TV: SATURDAY: 9:30-10:30AM, 1:00-3:30PM; 8:00-10:00 PM SUNDAY: 1:00-2:30PM and 8:00-10:00PM Ron watched sports programs for one half of the time. He watched movies for one third of the time. He watched cartoons with his sister Ronnette the rest of the time. How many minutes did Ron watch cartoons? Show all work:​

Answers

Ill try my best to "show my work" in text

First, we need to calculate the total amount of time Ron spent watching TV over the weekend.

On Saturday, he watched TV for 1 hour and 0 minutes from 9:30-10:30AM, 2 hours and 30 minutes from 1:00-3:30PM, and 2 hours from 8:00-10:00PM, for a total of 5 hours and 30 minutes on Saturday.

On Sunday, he watched TV for 1 hour and 30 minutes from 1:00-2:30PM, and 2 hours from 8:00-10:00PM, for a total of 2 hours and 30 minutes on Sunday.

The total time he spent watching TV over the weekend is 5 hours and 30 minutes on Saturday + 2 hours and 30 minutes on Sunday = 8 hours.

Next, we need to calculate the time he spent watching sports programs and movies.

He watched sports programs for half of the total time he spent watching TV, so he spent 8 hours / 2 = 4 hours watching sports programs.

He watched movies for one third of the total time he spent watching TV, so he spent 8 hours / 3 = 2 hours and 40 minutes watching movies.

Finally, we can calculate the time he spent watching cartoons with his sister.

He spent 8 hours watching TV total, and 4 hours watching sports programs and 2 hours and 40 minutes watching movies, so he spent 8 hours - 4 hours - 2 hours and 40 minutes = 1 hour and 20 minutes watching cartoons.

Since there are 60 minutes in an hour, the total time Ron spent watching cartoons is 1 hour and 20 minutes = 80 minutes.

Answer:

So Ron spent 1 hour and 40 minutes watching cartoons with his sister Ronnette.

Step-by-step explanation:

First, we need to calculate the total time that Ron spent watching TV. On Saturday, he watched TV for 1 hour in the morning, 2 hours and 30 minutes in the afternoon, and 2 hours in the evening, which adds up to 6 hours and 30 minutes. On Sunday, he watched TV for 1 hour and 30 minutes in the afternoon and 2 hours in the evening, which is a total of 3 hours and 30 minutes. So over the weekend, Ron watched TV for 6 hours and 30 minutes on Saturday and 3 hours and 30 minutes on Sunday, which adds up to a total of 10 hours.

Since Ron watched sports programs for half the time he watched TV, he spent 10 hours / 2 = 5 hours watching sports.

And since he watched movies for one-third of the time he watched TV, he spent 10 hours / 3 = 3 hours and 20 minutes watching movies.

Finally, we can calculate the time Ron spent watching cartoons by subtracting the time he spent watching sports and movies from the total time he spent watching TV:

10 hours - 5 hours - 3 hours and 20 minutes = 1 hour and 40 minutes

So Ron spent 1 hour and 40 minutes watching cartoons with his sister Ronnette.

Choose two equations that make up the system represented by the graph.
a. y= 5/4x
b. y=6-2.5x
c.y=2.5x + 6
d. y=6 - 3x
e. y=0.8x
Solve the system of equations and confirm the accuracy of your estimate

Answers

The system of equations are y = ( 5/4 )x and y = 6 - 3x where the value of x and y are 1.4117 and 1.765 respectively

What is an Equation of a line?

The equation of a line is expressed as y = mx + b where m is the slope and b is the y-intercept

And y - y₁ = m ( x - x₁ )

y = y-coordinate of second point

y₁ = y-coordinate of point one

m = slope

x = x-coordinate of second point

x₁ = x-coordinate of point one

The slope m = ( y₂ - y₁ ) / ( x₂ - x₁ )

Given data ,

Let the equation of line be represented as A

Now , the value of A is

Let the first equation be P

y = ( 5/4 )x   be equation (1)

Let the second equation be Q

y = 6 - 3x   be equation (2)

Substituting the values in the equation , we get

( 5/4 )x = 6 - 3x

Adding 3x on both sides of the equation , we get

6 = ( 5/4 + 3 ) x

( 17/4 )x = 6

Multiply by 4 on both sides of the equation , we get

17x = 24

Divide by 17 on both sides of the equation , we get

x = 24/17 = 1.4117

Substitute the value of x in equation (1) , we get

y = ( 5/4 ) ( 24/17 )

y = ( 30/17 )

y = 1.765

Hence , the equations are solved and the value of x and y are 24/17 and 30/17 respectively

To learn more about equation of line click :

https://brainly.com/question/14200719

#SPJ9

Find the value of the variable x. If your answer is not an integer, write it in simplest radical form with
the denominator rationalized h

Answers

The value of the variables y and z are 18

How to determine the value of the variables

From the question, we have the following parameters that can be used in our computation:

The special right triangle

The special right triangle has one of its angles to be 45 degrees

This means that

Legs = Hypotenuse/√2

Using the above as a guide, we have the following:

Legs = 18√2/√2

Evaluate

Legs = 18

Hence, the value of y and z are 18

Read more about right triangle at

https://brainly.com/question/2437195

#SPJ1

Dylan is hiking. He can either take a short cut up a steep flight of stairs or he can take the longer, flatter switchback route. If he goes on the switchback, he first travels west 60 meters and then turns and travels north 80 meters. Calculate the length in meters of the short cut up the flight of stairs. Round your answer to the nearest tenth.

Answers

We can use the Pythagorean theorem to find the length of the short cut up the flight of stairs. The theorem states that in a right triangle, the square of the length of the hypotenuse (the longest side) is equal to the sum of the squares of the lengths of the other two sides.

Let's call the length of the short cut "x". Then, we can set up the following equation:

x^2 = 60^2 + 80^2

Expanding the squares:

x^2 = 3600 + 6400

x^2 = 10000

Taking the square root of both sides:

x = sqrt(10000)

x = 100

So, the length of the short cut up the flight of stairs is 100 meters.

The area of a triangle is four times the area of a square. If the base of the triangle and a side of the square are equal, what is the ratio of the side of the square to the height of the triangle?.

Answers

Answer:

1:8

Step-by-step explanation:

since the base and sides are equal, I'll use 2 as a value for them. since all square sides are equal, we can find the area of the square. 2(2)=4. this means that in the situation the area of the triangle is 16.

16=1/2(2)h

32=2h

16=h

the height of the triangle is 16, while the side of the square is 2. the ratio is 2:16, or 1:8

Other Questions
If a business is expected to generate tax-favored income it may be better to choose the corporate business form.False.True 3. Do the calculation 43-113 using 2s complement. Your answer should be a signed binary number.4. Each of these 2s complement integers are stored in 6-bit computer words. Compute the sum as it is stored in a 6-bit computer word. Show the decimal equivalents of each operand and the sum. Indicate whether there is overflow. The foundation of a building is in the shape of a rectangle, with a length of 20 meters (m) and a width of 18 m. 1. Based on the passage what is the relationship between chemical energy, motion energy, and heat energy? 2. Based on the passage above draw your biomass pyramid below this time adding arrows to show the flow and relative amounts (%) of energy Convert 6 1/10 into a decimal. A scenario is the same as a(n) _____. a. use case b. relationship c. path through a use case d. collection of use cases e. role. Please Helppppp I dont understandddd If tan A=7/12, what is cos A? Use what you know about zeros of a function and end behavior of a graph to choose the graph that matches the function f(x)=(x-3)(x-1)(x+1). explain about wide stance lunges slight lean accounting for salaries expense without using a reversing entry requires the following on the date of payment: multiple choice question. compound entry that debits the expense and liability accounts and credits cash entry that debits the expense and credits the cash account for the same amounts entry that credits the expense and debits the cash account for the same amounts compound entry that credits the expense and liability accounts and debits cash PLEASE HELP ME ANSWER THIS QUESTION ASAP!! (Will mark as brainliest if correct and 50+ points!) How does paragraph 3 fit into the structure of Excerpt from A Legend of the Wooden Shoes? Use two details from the story to support your response. Scientists studying the blind cave-dwelling fish Astyanax mexicanus hypothesized that this fish evolved from a surface-dwelling fish. If this is true, which part of the anatomy of A. mexicanus is most likely to be a vestigial structure? Question 10 of 10In a command economy, decisions about which goods are produced arebased on:OA. what the government decides is important for societyOB. what goods are most likely to sell in international markets.C. what businesses believe will generate the most profits.OD. what the local community has made for generations.SUBMIT dr. obetz is conducting an experimental study on the effectiveness of antianxiety medication. which of the following will most likely be her independent variable? Find the difference. use your answer in the simplest terms, using the slash ( / ) as the fraction bar.1/2 - 1/4 find two consecutive numbers whose product is 272 what do scarcity mean in economic? Run tests can reveal non-random patterns in the time-ordered data. Select 4 patterns that might be seen.- Points outside limits- Trend- Scatter Plot- Cycles- Too much dispersion- Bias A certain reaction occurs in the following steps.(i) Cl(g) +O3(g) ClO (g) +O2(g)Identify the reaction intermediate and write the chemical equation for overall reaction.